1answer.
Ask question
Login Signup
Ask question
All categories
  • English
  • Mathematics
  • Social Studies
  • Business
  • History
  • Health
  • Geography
  • Biology
  • Physics
  • Chemistry
  • Computers and Technology
  • Arts
  • World Languages
  • Spanish
  • French
  • German
  • Advanced Placement (AP)
  • SAT
  • Medicine
  • Law
  • Engineering
hodyreva [135]
3 years ago
6

Use the diagram to solve for x.

Mathematics
1 answer:
Andreyy893 years ago
3 0
0 this is what I got
You might be interested in
The frequency of A4 is 440.00 Hz.<br> what is the frequency of A6, which is two octaves above A4?
zhannawk [14.2K]
To solve this problem you must apply the proccedure shown below:

 1. You have the following information given in the problem above: <span>The frequency of A4 is 440.00 Hz.

 2. So, you must apply the following formula to calculate the frequency of A6, which is two octaves above:

 (2</span>^n)f

 Where n is the octaves above and f is the known frequency.

 3. Therefore, you have:

 A6=(2^n)f
 A6=(2^2)(440.00 Hz)
 A6=1760 Hz

 Therefore, the answer is: A6=1760 Hz.
5 0
3 years ago
Read 2 more answers
20. Carmen can buy bottles of paint for $2.00 each and boxes of colored pencils for $3.50 each. She can spend no more than
EleoNora [17]

Answer:

(a) The equality that express many bottles of paint, x, and boxes of colored pencils, y, Carmen can buy is  2 x + 3.5 y = 42

b)Three different solutions:

x  = 14, y = 4 is First Solution.

x  = 7, y = 8 is Second Solution.

x  = 21, y = 0 is Third Solution.

Step-by-step explanation:

Here, the cost of 1 bottle of paint = $2.00 each

The cost of 1 box of colored pencils  = $3.5 each

Let us assume the number of bottle of paints purchased = x

So, the cost of x bottle of paints   = x ( Cost of 1 bottle of paint)

= x ($2.00)  = 2 x

Also, assume the number of box of colored pencils purchased = y

So, the cost of y box of colored pencils = y ( Cost of 1 box)

= y ($3.50)  = 3.5 y

Also, the total amount to be spent on art supplies  = $42

So, the total amount spent on x paint bottles + y box of colored pencil

= $42

or,  2 x + 3.5 y = 42

a ) So, the equality that express many bottles of paint, x, and boxes of colored pencils, y, Carmen can buy is  2 x + 3.5 y = 42

b)Three different solutions:

When y = 4 ,  equation is:   2 x + 3.5(4)  = 42

or,  2 x = 42 - 14  = 28, or x = 28/2  =  14

So, x  = 14, y = 4 is First Solution.

When y = 8 ,  equation is:   2 x + 3.5(8)  = 42

or,  2 x = 42 - 28  = 14, or x = 14/2  =  7

So, x  = 7, y = 8 is Second Solution.

When y = 0 ,  equation is:   2 x + 3.5(0)  = 42

or,  2 x = 42 , or x = 42/2  = 21

So, x  = 21, y = 0 is Third Solution.

7 0
3 years ago
What is the maxium number of right angles that can be contained in a triangle
alekssr [168]

Answer:

1

Step-by-step explanation:

A triangle can only have one right angle.

3 0
1 year ago
I need to know what 11, 14, and 15 are I don't remember how to do it
almond37 [142]

When you reflect a function in the <em>x</em>-axis, the first coordinate of a point stays the same, and the second coordinate changes sign (what was positive is now negative and vice versa).  See the attached picture.

Question 11:  f(x) = -5x + 2.  The function changes to its opposite, so g(x) = -(-5x + 2) = 5x - 2.

When you reflect a function in the <em>y</em>-axis, the first coordinate of a point changes to its opposite, but the second coordinate stays the same.  Replace  <em>x</em>  with  -<em>x</em> .

Question 14:  f(x) = |2x - 1| + 3.  Replacing <em>x</em> with -<em>x</em> produces  g(x) = |2(-x) - 1| + 3  which simplifies to  g(x) = |-2x -1| + 3.

Question 15 works the same way as #14.

6 0
2 years ago
Joe read 140 pages of his 300 page book what percent of his book has he read
kaheart [24]

Answer:

47

Step-by-step explanation:


8 0
3 years ago
Other questions:
  • There are 18 bulls and 45 cows on a ranch. If 4 more bulls and 4 more cows were added,will the ratio of bulls to cows remain the
    9·2 answers
  • An aircraft on a reconnaissance mission takes off from its home base and flies 550 miles at a bearing of S 46° E to a location i
    5·2 answers
  • There are a total of 56 students in the math club and the games club. The math club has 4 more students than the games club. Wri
    5·1 answer
  • What is the slope of the line between (-3,6) and (-5,9)?
    9·2 answers
  • Please help with #7! Thanks.
    7·1 answer
  • How many 3-digit hexadecimals start with a letter (A-F) or end with a numeral (0-9) (or both)? Explain.
    12·1 answer
  • Which one is linear and non linear and why
    12·1 answer
  • Someone help please
    13·1 answer
  • Can somebody help please?????????
    9·1 answer
  • Which of the following is true ?
    6·1 answer
Add answer
Login
Not registered? Fast signup
Signup
Login Signup
Ask question!